USMLE Forum - Largest USMLE Community

Full Version: easy one but high yield - darkhorse
You're currently viewing a stripped down version of our content. View the full version with proper formatting.
A 45-year-old man is brought to the emergency department after fainting while walking his dog. He denies a history of chest pain or prior syncope. His past medical history is significant for a cardiac murmur known since childhood. He does not smoke, and denies any recreational drug use. His temperature is 37 C (98.6 F), blood pressure is 128/78 mm Hg, pulse is 87 /min and regular, and respiratory rate is 18/min. He has a prominent apical impulse and a II/VI, late peaking murmur at the right upper sternal border that decreases in intensity with a Valsalva maneuver. An electrocardiogram shows large S waves in leads V1 and V2, and large R waves in leads V4 through V6, with ST depressions in leads V5 and V6. The most likely diagnosis is

A. aortic stenosis
B. idiopathic hypertrophic subaortic stenosis (hypertrophic cardiomyopathy)
C. pericarditis
D. pulmonary embolism
E. pulmonary hypertension
as means aortic stenosis
The correct answer is A. The presence of a systolic murmur over the aortic area that decreases as preload is decreased (i.e., with a Valsalva maneuver) is consistent with aortic stenosis. The fact that the murmur is late peaking suggests that the stenosis is significant. The prominent apical impulse and EKG suggests compensatory left ventricular hypertrophy (LVH). The ST changes in leads V5 and 6 are often seen with left ventricular hypertrophy from systolic overload, and are termed a œstrain pattern. The presence of syncope with aortic stenosis significantly impacts the mortality associated with this disease. Data suggests that the average time to death from aortic stenosis with syncope is 3 years.

Idiopathic hypertrophic subaortic stenosis (choice B) is unlikely since the murmur decreases with decreasing preload (the Valsalva maneuver). The outflow obstruction associated with idiopathic hypertrophic subaortic stenosis increases with decreasing preload and therefore, a decrease in preload is associated with an increase in the murmur intensity.

Pericarditis (choice C) is unlikely given the absence of chest pain, a precordial rub, or typical EKG findings (diffuse ST elevation, diffuse PR depression with PR elevation in aVL).

Pulmonary embolism (choice D) is unlikely given that the patient has no known history of risk factors for this process (e.g., hypercoagulability, poor activity), is not tachycardic or tachypneic, and has no EKG findings consistent with a pulmonary embolus (sinus tachycardia, S wave in lead I, Q wave and T wave inversion in lead III, or ST-T wave changes in leads V1 through V4).

Pulmonary hypertension (choice E) is unlikely given that there is no history of progressive dyspnea. The physical exam also does not show evidence of pulmonary hypertension (pulmonic tap, prominent pulmonic component of the second heart sound) or right heart strain (right ventricular heave, right sided S4, tricuspid regurgitation, etc).


A. aortic stenosis